Verbal questions from any Manhattan Prep GMAT Computer Adaptive Test. Topic subject should be the first few words of your question.
muralithe1
Students
 
Posts: 1
Joined: Sat Mar 06, 2010 9:35 am
 

To decrease the number of crimes in city Y, the city's Polic

by muralithe1 Sun Jul 04, 2010 12:40 am

To decrease the number of crimes in city Y, the city's Police Commissioner proposed taking some police officers from low-crime districts of the city and moving them to high-crime districts of the city. His proposal is based on city Y crime data that show that the number of crimes in any district of the city decreases when additional police officers are moved into that district.

The Police Commissioner's proposal depends on which of the following assumptions?

A.City X experienced a drastic reduction in crime after implementing a proposal similar to that proposed by the Police Commissioner of city Y.

B.The severity of crimes committed in any district of the city decreases when additional police officers are moved into that district.

C.The number of crimes committed in all high-crime districts of city Y is more than triple the number of crimes committed in all low-crime districts of city Y.

D.There are more low-crime districts than high-crime districts in city Y.

E.Districts of the city from which police officers are removed do not experience significant crime increases shortly after the removal of those officers.

The OA is E with the following official explanation.
The police commissioner's proposal would not make sense if districts of the city from which police officers are removed experience significant crime increases shortly after the removal of those officers. This would at least partially, if not fully, negate the reduction in the number of crimes in the high-crime districts. This choice establishes that, in fact, the low-crime districts do NOT suffer from significant crime increases after the removal of some officers--an essential assumption upon which the commissioner's proposal depends.

But mu doubt is why the commisioner has to be bothered about about the low-crime distircts after removal of officers from there.
In the statement he is bothered only about Y..Why we shld assume abt what will happen for low-crime districts after removal.

His proposal is based on city Y crime data that show that the number of crimes in any district of the city decreases when additional police officers are moved into that district.


Please correct me and clarify me....

Thanks,
Murali
tim
Course Students
 
Posts: 5665
Joined: Tue Sep 11, 2007 9:08 am
Location: Southwest Airlines, seat 21C
 

Re: To decrease the number of crimes in city Y, the city's Polic

by tim Sun Aug 01, 2010 1:08 pm

The reason we have to be concerned about the low-crime districts is because we are trying to lower the number of crimes city-wide. If we lower the number of crimes in one district, a corresponding increase in other districts would negate the benefit we were looking for..
Tim Sanders
Manhattan GMAT Instructor

Follow this link for some important tips to get the most out of your forum experience:
https://www.manhattanprep.com/gmat/forums/a-few-tips-t31405.html
Jov
Students
 
Posts: 23
Joined: Tue Sep 25, 2012 7:32 pm
 

Re: To decrease the number of crimes in city Y, the city's Polic

by Jov Mon Oct 27, 2014 12:47 am

For this question, I understand that option E is an answer choice which is perfectly valid as we look for necessary assumption.

I have few doubts related to answer choice A. In my understanding, there are three types of question:

Assumption question = Necessary assumption question (That assumption must be true in order to hold the conclusion)
Justify the conclusion question = Sufficient assumption question (With this assumption, the argument will be true but it must not to be true in order to hold the conclusion/ Strengthen the conclusion 100%)
Strengthen the conclusion = Any answer choice that strengthen the conclusion either 1% or 100% will be right.

Considering this, I think the first option strengthens the conclusion but does not justify the conclusion. Please let me know if my thinking is right or not.

Also I understood that Necessary and sufficient assumption questions are tested in LSAT exam. In my understanding that in GMAT all the assumption questions are Necessary assumption questions.Is it true that we don't need to study sufficient assumption concept as these type of questions are not tested in GMAT?
MayankS943
Students
 
Posts: 1
Joined: Sun Apr 10, 2016 5:39 am
 

Re: To decrease the number of crimes in city Y, the city's Polic

by MayankS943 Sun May 08, 2016 6:01 am

[size=100]Hello,
In the option E , there is "shortly" mentioned. Let's assume E to be correct . In the scenario crime rates increase after the "short" period , then commissioner's proposal fails because in his proposal there is no mention of short term crime fall or rise. The closest answer in that case seems to be option D.
[/size]
ajaym8
Students
 
Posts: 24
Joined: Tue Apr 12, 2016 7:32 pm
 

Re: To decrease the number of crimes in city Y, the city's Polic

by ajaym8 Sun Jan 01, 2017 3:23 pm

MayankS943 Wrote:[size=100]Hello,
In the option E , there is "shortly" mentioned. Let's assume E to be correct . In the scenario crime rates increase after the "short" period , then commissioner's proposal fails because in his proposal there is no mention of short term crime fall or rise. The closest answer in that case seems to be option D.
[/size]


I have 2 points with regard to your query (instructors, please correct me if I am wrong).:

1. The case presented in E is that movement of officers from low crime districts(LCD) to high crime districts(HCD) will not affect the districts they are leaving. This is the assumption made by the author in making the argument airtight. Another , similar assumption could be that the LCD always have enough officers in reserves that movement of any number of officers out of those LCDs will not affect the crime rate in those LCDs . "Shortly" is used to signify a time frame that is not relevant for us. Whether LCD experiences higher crime after movement "shortly" or "after a long time" is not our concern. Question is whether LCDs will experience change in crime rate due to proposed movement.

2. Answer D plays on the number of LCD & HCD in Y. How does that count matter ? I hope you are not assuming that LCDs have less officers than HCDs.
RonPurewal
Students
 
Posts: 19744
Joined: Tue Aug 14, 2007 8:23 am
 

Re: To decrease the number of crimes in city Y, the city's Polic

by RonPurewal Mon Jan 09, 2017 5:28 am

MayankS943 Wrote:[size=100]Hello,
In the option E , there is "shortly" mentioned. Let's assume E to be correct . In the scenario crime rates increase after the "short" period , then commissioner's proposal fails because in his proposal there is no mention of short term crime fall or rise. The closest answer in that case seems to be option D.
[/size]


^^ "shortly" doesn't matter much, although it makes the statement even more specific/narrow in scope (...which makes it a better answer to a question like this one).

also, you're claiming that the answer should be D, but you haven't offered any support for that answer choice. why do you think D is correct?
RonPurewal
Students
 
Posts: 19744
Joined: Tue Aug 14, 2007 8:23 am
 

Re: To decrease the number of crimes in city Y, the city's Polic

by RonPurewal Mon Jan 09, 2017 5:30 am

ajaym8, do you have a question of your own? if so, please clarify; thank you.

please do not attempt to answer other students' questions.
this isn't that kind of forum. on this forum, only moderators provide answers.
thank you.
ajaym8
Students
 
Posts: 24
Joined: Tue Apr 12, 2016 7:32 pm
 

Re: To decrease the number of crimes in city Y, the city's Polic

by ajaym8 Mon Jan 09, 2017 6:41 am

RonPurewal Wrote:ajaym8, do you have a question of your own? if so, please clarify; thank you.

please do not attempt to answer other students' questions.
this isn't that kind of forum. on this forum, only moderators provide answers.
thank you.


Alright. I didn't know that. I have no questions for this thread.
I'll take care in future.

Thanks,
RonPurewal
Students
 
Posts: 19744
Joined: Tue Aug 14, 2007 8:23 am
 

Re: To decrease the number of crimes in city Y, the city's Polic

by RonPurewal Sun Jan 22, 2017 1:02 pm

appreciated.